LSAT and Law School Admissions Forum

Get expert LSAT preparation and law school admissions advice from PowerScore Test Preparation.

User avatar
 Dave Killoran
PowerScore Staff
  • PowerScore Staff
  • Posts: 5852
  • Joined: Mar 25, 2011
|
#46196
Complete Question Explanation
(The complete setup for this game can be found here: lsat/viewtopic.php?t=6424)

The correct answer choice is (E)

Answer choice (A) can be eliminated because according to the third rule F must be interviewed.

Answer choice (B) can be eliminated because from the second rule when J is interviewed, L must also be interviewed.

Answer choices (C) and (D) can be eliminated because when G is interviewed, both J and I must also be interviewed. Since F is always interviewed, when G is interviewed at least three other applicants must also be interviewed.

Answer choice (E) is thus proven correct by process of elimination.

Get the most out of your LSAT Prep Plus subscription.

Analyze and track your performance with our Testing and Analytics Package.